Difference between revisions of "1983 AHSME Problems/Problem 20"
Katzrockso (talk | contribs) (Created page with "== Problem 20 == If <math>\tan{\alpha}</math> and <math>\tan{\beta}</math> are the roots of <math>x^2 - px + q = 0</math>, and <math>\cot{\alpha}</math> and <math>\cot{\beta}...") |
Sevenoptimus (talk | contribs) m (Fixed spacing) |
||
(4 intermediate revisions by 3 users not shown) | |||
Line 4: | Line 4: | ||
are the roots of <math>x^2 - rx + s = 0</math>, then <math>rs</math> is necessarily | are the roots of <math>x^2 - rx + s = 0</math>, then <math>rs</math> is necessarily | ||
− | <math>\ | + | <math>\textbf{(A)} \ pq \qquad |
− | \ | + | \textbf{(B)} \ \frac{1}{pq} \qquad |
− | \ | + | \textbf{(C)} \ \frac{p}{q^2} \qquad |
− | \ | + | \textbf{(D)}\ \frac{q}{p^2}\qquad |
− | \ | + | \textbf{(E)}\ \frac{p}{q}</math> |
== Solution == | == Solution == | ||
− | By Vieta's | + | By Vieta's Formulae, we have <math>\tan(\alpha)\tan(\beta)=q</math> and <math>\cot(\alpha)\cot(\beta)=s</math>. Recalling that <math>\cot\theta=\frac{1}{\tan\theta}</math>, we have <math>\frac{1}{\tan(\alpha)\tan(\beta)}=\frac{1}{q}=s</math>. |
− | + | Also by Vieta's Formulae, we have <math>\tan(\alpha)+\tan(\beta)=p</math> and <math>\cot(\alpha)+\cot(\beta)=r</math>, and again using <math>\cot\theta=\frac{1}{\tan\theta}</math>, we have <math>\tan(\alpha)+\tan(\beta)=r(\tan(\alpha)\tan(\beta))</math>. Using <math>\tan(\alpha)\tan(\beta)=q</math> and <math>\tan(\alpha)+\tan(\beta)=p</math>, we therefore deduce that <math>r=\frac{p}{q}</math>, which yields <math>rs = \frac{1}{q}\cdot\frac{p}{q}=\frac{p}{q^2}</math>. | |
− | Thus, the answer is <math>(C) \frac{p}{q^2}</math> | + | Thus, the answer is <math>\boxed{\textbf{(C)} \ \frac{p}{q^2}}</math>. |
+ | |||
+ | ==See Also== | ||
+ | {{AHSME box|year=1983|num-b=19|num-a=21}} | ||
+ | |||
+ | {{MAA Notice}} |
Latest revision as of 23:58, 19 February 2019
Problem 20
If and are the roots of , and and are the roots of , then is necessarily
Solution
By Vieta's Formulae, we have and . Recalling that , we have .
Also by Vieta's Formulae, we have and , and again using , we have . Using and , we therefore deduce that , which yields .
Thus, the answer is .
See Also
1983 AHSME (Problems • Answer Key • Resources) | ||
Preceded by Problem 19 |
Followed by Problem 21 | |
1 • 2 • 3 • 4 • 5 • 6 • 7 • 8 • 9 • 10 • 11 • 12 • 13 • 14 • 15 • 16 • 17 • 18 • 19 • 20 • 21 • 22 • 23 • 24 • 25 • 26 • 27 • 28 • 29 • 30 | ||
All AHSME Problems and Solutions |
The problems on this page are copyrighted by the Mathematical Association of America's American Mathematics Competitions.